Difference between revisions of "2023 WSMO Team Round Problems/Problem 2"

(Created page with "==Problem== Integers <math>W,S,M,</math> and <math>O</math> satisfy <math>W+S+M+O = 13.</math> Find the maximum possible value of <math>WSMO.</math> ==Solution==")
 
 
Line 4: Line 4:
  
 
==Solution==
 
==Solution==
 +
From the AM-GM inequality, we want <math>W,S,M,O</math> as close to each other as possible. So, let <math>W=S=M=3,O=4\implies WSMO = \boxed{108}.</math>
 +
 +
~pinkpig

Latest revision as of 13:50, 13 September 2025

Problem

Integers $W,S,M,$ and $O$ satisfy $W+S+M+O = 13.$ Find the maximum possible value of $WSMO.$

Solution

From the AM-GM inequality, we want $W,S,M,O$ as close to each other as possible. So, let $W=S=M=3,O=4\implies WSMO = \boxed{108}.$

~pinkpig